Remainders

This topic has expert replies
User avatar
Senior | Next Rank: 100 Posts
Posts: 58
Joined: Thu Jul 22, 2010 5:40 pm
Thanked: 1 times

Remainders

by Jinglander » Sun Nov 21, 2010 7:58 am
This is from memory so it may be non exact. If k^4 is divisible by 81 then which of the following could be the remainder when k is divided by 9. I think the answer choices were 1,2,4,6,8
Last edited by Jinglander on Sun Nov 21, 2010 8:22 am, edited 1 time in total.

Master | Next Rank: 500 Posts
Posts: 358
Joined: Fri Jun 25, 2010 9:47 am
Location: Mumbai
Thanked: 34 times
Followed by:4 members
GMAT Score:740

by karanrulz4ever » Sun Nov 21, 2010 8:19 am
Well u r not supposed to reveal the actual GMAT questions...did u read the non-disclosure policy of GMAT?

The answer should be 6 btw...

User avatar
Senior | Next Rank: 100 Posts
Posts: 58
Joined: Thu Jul 22, 2010 5:40 pm
Thanked: 1 times

by Jinglander » Sun Nov 21, 2010 8:23 am
Can you show steps towards solution

Legendary Member
Posts: 1119
Joined: Fri May 07, 2010 8:50 am
Thanked: 29 times
Followed by:3 members

by diebeatsthegmat » Sun Nov 21, 2010 6:15 pm
Jinglander wrote:Can you show steps towards solution
how can we show you the solution while we dont understand what the problem is about.... :(

User avatar
Legendary Member
Posts: 866
Joined: Mon Aug 02, 2010 6:46 pm
Location: Gwalior, India
Thanked: 31 times

by goyalsau » Sun Nov 21, 2010 9:19 pm
Jinglander wrote:This is from memory so it may be non exact. If k^4 is divisible by 81 then which of the following could be the remainder when k is divided by 9. I think the answer choices were 1,2,4,6,8
Though I am also not able to understand the problem But i thought there is no harm in giving a try,

K^4 is divisible by 81 ,

It means K must have a least one 3 in its factor, Because only then 3 ^ 4 will be divisible by 81.

Now k ^ 2 must have been divisible by 9 { Because at least two 3's are there in it }

Now K must have at least one 3 in it.

Now if we write first 10 multiples of 3 , it will be

0
3
6
9
12
15
18
21
24
27
30

Now 0 / 9 ===Remainder 0
3 / 9 ===remainder will be 3
6 / 9 ===remainder will be 6
9 / 9 ===remainder will be 0
12/9 ===remainder will be 3
15 / 9 ===remainder will be 6
18 / 9 === remainder will be 0
21/9 ===remainder will be 0

like this it will be continue...

remainders will be in the form of 0 , 3 , 6 , 0 , 3 , 6 ........................

we don't have 0 3 in the answer choice so my take would be 6 ...................
Saurabh Goyal
[email protected]
-------------------------


EveryBody Wants to Win But Nobody wants to prepare for Win.

User avatar
Legendary Member
Posts: 866
Joined: Mon Aug 02, 2010 6:46 pm
Location: Gwalior, India
Thanked: 31 times

by goyalsau » Sun Nov 21, 2010 9:21 pm
karanrulz4ever wrote:Well u r not supposed to reveal the actual GMAT questions...did u read the non-disclosure policy of GMAT?
Common Buddy He is not going to jail for revealing the Actual GMAT question ...................
Saurabh Goyal
[email protected]
-------------------------


EveryBody Wants to Win But Nobody wants to prepare for Win.